Das Wasserstoffatom (relativistsich): Unterschied zwischen den Versionen

Aus PhysikWiki
Zur Navigation springen Zur Suche springen
Keine Bearbeitungszusammenfassung
*>SchuBot
Einrückungen Mathematik
Zeile 3: Zeile 3:
In einem rotationssymmetrischen Potenzial haben wir als Dirac- Hamiltonian:
In einem rotationssymmetrischen Potenzial haben wir als Dirac- Hamiltonian:


<math>\begin{align}
:<math>\begin{align}


& H=\left( c\bar{\alpha }\bar{p}+{{m}_{0}}{{c}^{2}}\beta +V(r) \right) \\
& H=\left( c\bar{\alpha }\bar{p}+{{m}_{0}}{{c}^{2}}\beta +V(r) \right) \\
Zeile 21: Zeile 21:
Man kann den Hamilton- Operator schreiben als:
Man kann den Hamilton- Operator schreiben als:


<math>H=\left( c{{\alpha }_{r}}{{p}_{r}}+\frac{ic}{r}{{\alpha }_{r}}\beta \hbar Q+{{m}_{0}}{{c}^{2}}\beta +V(r) \right)</math>
:<math>H=\left( c{{\alpha }_{r}}{{p}_{r}}+\frac{ic}{r}{{\alpha }_{r}}\beta \hbar Q+{{m}_{0}}{{c}^{2}}\beta +V(r) \right)</math>


Beweis:
Beweis:


<math>\begin{align}
:<math>\begin{align}


& {{\alpha }_{r}}{{p}_{r}}+\frac{i}{r}{{\alpha }_{r}}\beta \hbar Q={{\alpha }_{r}}\left[ \frac{1}{r}\left( \bar{r}\bar{p}-i\hbar  \right)+\frac{i}{r}{{\beta }^{2}}\left( \tilde{\bar{\sigma }}\bar{L}+\hbar  \right) \right] \\
& {{\alpha }_{r}}{{p}_{r}}+\frac{i}{r}{{\alpha }_{r}}\beta \hbar Q={{\alpha }_{r}}\left[ \frac{1}{r}\left( \bar{r}\bar{p}-i\hbar  \right)+\frac{i}{r}{{\beta }^{2}}\left( \tilde{\bar{\sigma }}\bar{L}+\hbar  \right) \right] \\
Zeile 41: Zeile 41:
Es gilt weiter:
Es gilt weiter:


<math>\left[ \hbar Q,H \right]=0</math>
:<math>\left[ \hbar Q,H \right]=0</math>


. Somit existieren gemeinsame Eigenzustände zu <math>H</math>
. Somit existieren gemeinsame Eigenzustände zu <math>H</math>
Zeile 51: Zeile 51:
:
:


<math>\begin{align}
:<math>\begin{align}


& {{\left( \hbar Q \right)}^{2}}=\beta \left( \tilde{\bar{\sigma }}\bar{L}+\hbar  \right)\beta \left( \tilde{\bar{\sigma }}\bar{L}+\hbar  \right)={{\beta }^{2}}{{\left( \tilde{\bar{\sigma }}\bar{L}+\hbar  \right)}^{2}} \\
& {{\left( \hbar Q \right)}^{2}}=\beta \left( \tilde{\bar{\sigma }}\bar{L}+\hbar  \right)\beta \left( \tilde{\bar{\sigma }}\bar{L}+\hbar  \right)={{\beta }^{2}}{{\left( \tilde{\bar{\sigma }}\bar{L}+\hbar  \right)}^{2}} \\
Zeile 77: Zeile 77:
Somit:
Somit:


<math>\begin{align}
:<math>\begin{align}


& {{\left( \hbar Q \right)}^{2}}={{L}^{2}}+\hbar \tilde{\bar{\sigma }}\bar{L}+{{\hbar }^{2}}={{\left( \bar{L}+\frac{\hbar }{2}\tilde{\bar{\sigma }} \right)}^{2}}+\frac{{{\hbar }^{2}}}{4} \\
& {{\left( \hbar Q \right)}^{2}}={{L}^{2}}+\hbar \tilde{\bar{\sigma }}\bar{L}+{{\hbar }^{2}}={{\left( \bar{L}+\frac{\hbar }{2}\tilde{\bar{\sigma }} \right)}^{2}}+\frac{{{\hbar }^{2}}}{4} \\
Zeile 91: Zeile 91:
Schließlich also
Schließlich also


<math>{{\left( \hbar Q \right)}^{2}}={{\bar{J}}^{2}}+\frac{{{\hbar }^{2}}}{4}</math>
:<math>{{\left( \hbar Q \right)}^{2}}={{\bar{J}}^{2}}+\frac{{{\hbar }^{2}}}{4}</math>


Die Eigenwerte von <math>{{\bar{J}}^{2}}</math>
Die Eigenwerte von <math>{{\bar{J}}^{2}}</math>
Zeile 99: Zeile 99:
mit <math>j=l\pm s=\frac{1}{2},\frac{3}{2},...</math>
mit <math>j=l\pm s=\frac{1}{2},\frac{3}{2},...</math>


<math>\begin{align}
:<math>\begin{align}


& {{\left( \hbar Q \right)}^{2}}\left| j \right\rangle =\left( {{\hbar }^{2}}j(j+1)+\frac{{{\hbar }^{2}}}{4} \right)\left| j \right\rangle ={{\hbar }^{2}}{{(j+\frac{1}{2})}^{2}}\left| j \right\rangle  \\
& {{\left( \hbar Q \right)}^{2}}\left| j \right\rangle =\left( {{\hbar }^{2}}j(j+1)+\frac{{{\hbar }^{2}}}{4} \right)\left| j \right\rangle ={{\hbar }^{2}}{{(j+\frac{1}{2})}^{2}}\left| j \right\rangle  \\
Zeile 109: Zeile 109:
Somit:
Somit:


<math>\begin{align}
:<math>\begin{align}


& \left( \hbar Q \right)\left| j \right\rangle =\left( \hbar q \right)\left| j \right\rangle  \\
& \left( \hbar Q \right)\left| j \right\rangle =\left( \hbar q \right)\left| j \right\rangle  \\
Zeile 119: Zeile 119:
Es bleibt das radiale Eigenwertproblem für
Es bleibt das radiale Eigenwertproblem für


<math>H=\left( c{{\alpha }_{r}}{{p}_{r}}+\frac{ic}{r}{{\alpha }_{r}}\beta \hbar Q+{{m}_{0}}{{c}^{2}}\beta +V(r) \right)</math>
:<math>H=\left( c{{\alpha }_{r}}{{p}_{r}}+\frac{ic}{r}{{\alpha }_{r}}\beta \hbar Q+{{m}_{0}}{{c}^{2}}\beta +V(r) \right)</math>


'''Geeignete Darstellung für '''<math>{{\alpha }_{r}}</math>
'''Geeignete Darstellung für '''<math>{{\alpha }_{r}}</math>
Zeile 125: Zeile 125:
:
:


<math>\begin{align}
:<math>\begin{align}


& {{\left( {{\alpha }_{r}} \right)}^{2}}=\frac{1}{{{r}^{2}}}\left( \bar{\alpha }\bar{r} \right)\left( \bar{\alpha }\bar{r} \right)=\frac{1}{{{r}^{2}}}{{\alpha }^{\mu }}{{\alpha }^{\nu }}{{x}^{\mu }}{{x}^{\nu }}=\frac{1}{2{{r}^{2}}}\left( {{\alpha }^{\mu }}{{\alpha }^{\nu }}+{{\alpha }^{\nu }}{{\alpha }^{\mu }} \right){{x}^{\mu }}{{x}^{\nu }} \\
& {{\left( {{\alpha }_{r}} \right)}^{2}}=\frac{1}{{{r}^{2}}}\left( \bar{\alpha }\bar{r} \right)\left( \bar{\alpha }\bar{r} \right)=\frac{1}{{{r}^{2}}}{{\alpha }^{\mu }}{{\alpha }^{\nu }}{{x}^{\mu }}{{x}^{\nu }}=\frac{1}{2{{r}^{2}}}\left( {{\alpha }^{\mu }}{{\alpha }^{\nu }}+{{\alpha }^{\nu }}{{\alpha }^{\mu }} \right){{x}^{\mu }}{{x}^{\nu }} \\
Zeile 141: Zeile 141:
Für
Für


<math>\beta =\left( \begin{matrix}
:<math>\beta =\left( \begin{matrix}


1 & 0  \\
1 & 0  \\
Zeile 161: Zeile 161:
erfüllt werden:
erfüllt werden:


<math>\begin{align}
:<math>\begin{align}


& {{\alpha }_{r}}\beta =\left( \begin{matrix}
& {{\alpha }_{r}}\beta =\left( \begin{matrix}
Zeile 183: Zeile 183:
Es gilt:
Es gilt:


<math>\begin{align}
:<math>\begin{align}


& {{p}_{r}}=\frac{1}{r}\left( \bar{r}\bar{p}-i\hbar  \right) \\
& {{p}_{r}}=\frac{1}{r}\left( \bar{r}\bar{p}-i\hbar  \right) \\
Zeile 195: Zeile 195:
Also
Also


<math>H=\hbar c\left( \begin{matrix}
:<math>H=\hbar c\left( \begin{matrix}


0 & -1  \\
0 & -1  \\
Zeile 223: Zeile 223:
Ansatz für den Radialanteil
Ansatz für den Radialanteil


<math>\left( \begin{matrix}
:<math>\left( \begin{matrix}


{{\phi }_{a}}  \\
{{\phi }_{a}}  \\
Zeile 239: Zeile 239:
Eingesetzt in die Eigenwertgleichung für H:
Eingesetzt in die Eigenwertgleichung für H:


<math>\left( \begin{matrix}
:<math>\left( \begin{matrix}


{{\phi }_{a}}  \\
{{\phi }_{a}}  \\
Zeile 255: Zeile 255:
folgt:
folgt:


<math>\begin{align}
:<math>\begin{align}


& -\frac{\hbar c}{r}\frac{dG}{dr}-\frac{c\hbar q}{{{r}^{2}}}G+\frac{{{m}_{0}}{{c}^{2}}}{r}F+\frac{V}{r}F=E\frac{F}{r} \\
& -\frac{\hbar c}{r}\frac{dG}{dr}-\frac{c\hbar q}{{{r}^{2}}}G+\frac{{{m}_{0}}{{c}^{2}}}{r}F+\frac{V}{r}F=E\frac{F}{r} \\
Zeile 267: Zeile 267:
Also:
Also:


<math>\begin{align}
:<math>\begin{align}


& \left( E-{{m}_{0}}{{c}^{2}}-V \right)F+\hbar c\frac{dG}{dr}+\frac{c\hbar q}{r}G=0 \\
& \left( E-{{m}_{0}}{{c}^{2}}-V \right)F+\hbar c\frac{dG}{dr}+\frac{c\hbar q}{r}G=0 \\
Zeile 278: Zeile 278:


====Skalentransformation:====
====Skalentransformation:====
<math>\begin{align}
:<math>\begin{align}


& {{a}_{1}}=\frac{{{m}_{0}}{{c}^{2}}+E}{\hbar c} \\
& {{a}_{1}}=\frac{{{m}_{0}}{{c}^{2}}+E}{\hbar c} \\
Zeile 290: Zeile 290:
Führt man des weiteren ein:
Führt man des weiteren ein:


<math>\begin{align}
:<math>\begin{align}


& \rho :=ar \\
& \rho :=ar \\
Zeile 302: Zeile 302:
wodurch sich auch das Potenzial vereinfacht zu:
wodurch sich auch das Potenzial vereinfacht zu:


<math>\frac{V}{\hbar ca}=-\frac{\gamma }{\rho }</math>
:<math>\frac{V}{\hbar ca}=-\frac{\gamma }{\rho }</math>


:
:


<math>\begin{align}
:<math>\begin{align}


& \left( \frac{d}{d\rho }+\frac{q}{\rho } \right)G-\left( \frac{{{a}_{2}}}{a}-\frac{\gamma }{\rho } \right)F=0 \\
& \left( \frac{d}{d\rho }+\frac{q}{\rho } \right)G-\left( \frac{{{a}_{2}}}{a}-\frac{\gamma }{\rho } \right)F=0 \\
Zeile 316: Zeile 316:
<u>'''Randbedingung:'''</u>
<u>'''Randbedingung:'''</u>


<math>F(\rho ),G(\rho )</math>
:<math>F(\rho ),G(\rho )</math>


regulär bei <math>\rho \to 0</math>
regulär bei <math>\rho \to 0</math>


<math>F(\rho ),G(\rho )\to 0</math>
:<math>F(\rho ),G(\rho )\to 0</math>


für <math>\rho \to \infty </math>
für <math>\rho \to \infty </math>
Zeile 336: Zeile 336:
'''Asymptotisches Verhalten:'''
'''Asymptotisches Verhalten:'''


<math>\begin{align}
:<math>\begin{align}


& \rho \to \infty  \\
& \rho \to \infty  \\
Zeile 352: Zeile 352:
divergiert !
divergiert !


<math>\begin{align}
:<math>\begin{align}


& \rho \to 0 \\
& \rho \to 0 \\
Zeile 364: Zeile 364:
Ansatz:
Ansatz:


<math>\begin{align}
:<math>\begin{align}


& F(\rho )={{f}_{0}}{{\rho }^{\lambda }} \\
& F(\rho )={{f}_{0}}{{\rho }^{\lambda }} \\
Zeile 386: Zeile 386:
Ansatz:
Ansatz:


<math>\begin{align}
:<math>\begin{align}


& F(\rho )={{\rho }^{\lambda }}{{e}^{-\rho }}f\left( \rho  \right) \\
& F(\rho )={{\rho }^{\lambda }}{{e}^{-\rho }}f\left( \rho  \right) \\
Zeile 400: Zeile 400:
Die Lösung erfolgt über einen Potenzreihenansatz:
Die Lösung erfolgt über einen Potenzreihenansatz:


<math>\begin{align}
:<math>\begin{align}


& f(\rho )=\sum\limits_{k=0}^{\infty }{{{f}_{k}}{{\rho }^{k}}}\Rightarrow f\acute{\ }(\rho )=\sum\limits_{k=1}^{\infty }{k{{f}_{k}}{{\rho }^{k-1}}}=\sum\limits_{k=0}^{\infty }{(k+1){{f}_{k+1}}{{\rho }^{k}}} \\
& f(\rho )=\sum\limits_{k=0}^{\infty }{{{f}_{k}}{{\rho }^{k}}}\Rightarrow f\acute{\ }(\rho )=\sum\limits_{k=1}^{\infty }{k{{f}_{k}}{{\rho }^{k-1}}}=\sum\limits_{k=0}^{\infty }{(k+1){{f}_{k+1}}{{\rho }^{k}}} \\
Zeile 416: Zeile 416:
Koeffizientenvergleich liefert:
Koeffizientenvergleich liefert:


<math>\begin{align}
:<math>\begin{align}


& O\left( \frac{1}{\rho } \right):\left( \lambda +q \right){{g}_{0}}+\gamma {{f}_{0}}=0\quad \quad \left( \lambda -q \right){{f}_{0}}-\gamma {{g}_{0}}=0 \\
& O\left( \frac{1}{\rho } \right):\left( \lambda +q \right){{g}_{0}}+\gamma {{f}_{0}}=0\quad \quad \left( \lambda -q \right){{f}_{0}}-\gamma {{g}_{0}}=0 \\
Zeile 426: Zeile 426:
bis auf Normfaktor
bis auf Normfaktor


<math>\begin{align}
:<math>\begin{align}


& O\left( {{\rho }^{k}} \right):\left( \lambda +q+k+1 \right){{g}_{k+1}}-{{g}_{k}}+\gamma {{f}_{k+1}}-\frac{{{a}_{2}}}{a}{{f}_{k}}=0 \\
& O\left( {{\rho }^{k}} \right):\left( \lambda +q+k+1 \right){{g}_{k+1}}-{{g}_{k}}+\gamma {{f}_{k+1}}-\frac{{{a}_{2}}}{a}{{f}_{k}}=0 \\
Zeile 436: Zeile 436:
k=0,1,2,....  Rekursionsformel !!
k=0,1,2,....  Rekursionsformel !!


<math>\begin{align}
:<math>\begin{align}


& a\left[ \left( \lambda +q+k+1 \right){{g}_{k+1}}-{{g}_{k}}+\gamma {{f}_{k+1}}-\frac{{{a}_{2}}}{a}{{f}_{k}} \right]-{{a}_{2}}\left[ \left( \lambda -q+k+1 \right){{f}_{k+1}}-{{f}_{k}}+\gamma {{g}_{k+1}}-\frac{{{a}_{1}}}{a}{{g}_{k}} \right]=0 \\
& a\left[ \left( \lambda +q+k+1 \right){{g}_{k+1}}-{{g}_{k}}+\gamma {{f}_{k+1}}-\frac{{{a}_{2}}}{a}{{f}_{k}} \right]-{{a}_{2}}\left[ \left( \lambda -q+k+1 \right){{f}_{k+1}}-{{f}_{k}}+\gamma {{g}_{k+1}}-\frac{{{a}_{1}}}{a}{{g}_{k}} \right]=0 \\
Zeile 446: Zeile 446:
'''Verhalten für große k:'''
'''Verhalten für große k:'''


<math>ak{{g}_{k+1}}\approx {{a}_{2}}k{{f}_{k+1}}\Rightarrow {{f}_{k}}\approx \frac{a}{{{a}_{2}}}{{g}_{k}}</math>
:<math>ak{{g}_{k+1}}\approx {{a}_{2}}k{{f}_{k+1}}\Rightarrow {{f}_{k}}\approx \frac{a}{{{a}_{2}}}{{g}_{k}}</math>


Dies kann man einsetzen in
Dies kann man einsetzen in


<math>\left( \lambda +q+k+1 \right){{g}_{k+1}}-{{g}_{k}}+\gamma {{f}_{k+1}}-\frac{{{a}_{2}}}{a}{{f}_{k}}=0</math>
:<math>\left( \lambda +q+k+1 \right){{g}_{k+1}}-{{g}_{k}}+\gamma {{f}_{k+1}}-\frac{{{a}_{2}}}{a}{{f}_{k}}=0</math>


und es folgt:
und es folgt:


<math>\begin{align}
:<math>\begin{align}


& \left( k+1 \right){{g}_{k+1}}\approx 2{{g}_{k}} \\
& \left( k+1 \right){{g}_{k+1}}\approx 2{{g}_{k}} \\
Zeile 468: Zeile 468:
Falls die Potenzreihen
Falls die Potenzreihen


<math>f(\rho )=\sum\limits_{k=0}^{\infty }{{{f}_{k}}{{\rho }^{k}}},g(\rho )=\sum\limits_{k=0}^{\infty }{{{g}_{k}}{{\rho }^{k}}}</math>
:<math>f(\rho )=\sum\limits_{k=0}^{\infty }{{{f}_{k}}{{\rho }^{k}}},g(\rho )=\sum\limits_{k=0}^{\infty }{{{g}_{k}}{{\rho }^{k}}}</math>


nicht abbrechen, so divergiert <math>\begin{align}
nicht abbrechen, so divergiert <math>\begin{align}
Zeile 486: Zeile 486:
geben:
geben:


<math>{{f}_{n\acute{\ }+1}}={{g}_{n\acute{\ }+1}}=0</math>
:<math>{{f}_{n\acute{\ }+1}}={{g}_{n\acute{\ }+1}}=0</math>


Setzt man dies in die Rekursionsformel ein, so folgt:
Setzt man dies in die Rekursionsformel ein, so folgt:


<math>\begin{align}
:<math>\begin{align}


& -{{g}_{n\acute{\ }}}-\frac{{{a}_{2}}}{a}{{f}_{n\acute{\ }}}=0\Rightarrow {{a}_{2}}{{f}_{n\acute{\ }}}=-a{{g}_{n\acute{\ }}} \\
& -{{g}_{n\acute{\ }}}-\frac{{{a}_{2}}}{a}{{f}_{n\acute{\ }}}=0\Rightarrow {{a}_{2}}{{f}_{n\acute{\ }}}=-a{{g}_{n\acute{\ }}} \\
Zeile 500: Zeile 500:
Diese beiden Gleichungen stimmen jedoch für alle f,g überein, da
Diese beiden Gleichungen stimmen jedoch für alle f,g überein, da


<math>\frac{{{a}_{2}}}{a}=\frac{a}{{{a}_{1}}}</math>
:<math>\frac{{{a}_{2}}}{a}=\frac{a}{{{a}_{1}}}</math>


Setzt man <math>{{a}_{2}}{{f}_{n\acute{\ }}}=-a{{g}_{n\acute{\ }}}</math>
Setzt man <math>{{a}_{2}}{{f}_{n\acute{\ }}}=-a{{g}_{n\acute{\ }}}</math>
Zeile 510: Zeile 510:
:
:


<math>\begin{align}
:<math>\begin{align}


& \frac{a\left( \lambda +q+n\acute{\ } \right)+{{a}_{2}}\gamma }{a}=-\frac{\left[ {{a}_{2}}\left( \lambda -q+n\acute{\ } \right)-a\gamma  \right]}{{{a}_{2}}} \\
& \frac{a\left( \lambda +q+n\acute{\ } \right)+{{a}_{2}}\gamma }{a}=-\frac{\left[ {{a}_{2}}\left( \lambda -q+n\acute{\ } \right)-a\gamma  \right]}{{{a}_{2}}} \\
Zeile 526: Zeile 526:
Weiter gilt:
Weiter gilt:


<math>\begin{align}
:<math>\begin{align}


& {{a}^{2}}=\frac{{{m}_{0}}^{2}{{c}^{4}}-{{E}^{2}}}{{{\hbar }^{2}}{{c}^{2}}} \\
& {{a}^{2}}=\frac{{{m}_{0}}^{2}{{c}^{4}}-{{E}^{2}}}{{{\hbar }^{2}}{{c}^{2}}} \\
Zeile 536: Zeile 536:
Löst man dies nach den exakten Energieeigenwerten, die sich damit ergeben, also nach E auf, so erhält man die Feinstrukturformel:
Löst man dies nach den exakten Energieeigenwerten, die sich damit ergeben, also nach E auf, so erhält man die Feinstrukturformel:


<math>E=\frac{{{m}_{0}}{{c}^{2}}}{\sqrt{1+{{\left( \frac{\gamma }{\lambda +n\acute{\ }} \right)}^{2}}}}</math>
:<math>E=\frac{{{m}_{0}}{{c}^{2}}}{\sqrt{1+{{\left( \frac{\gamma }{\lambda +n\acute{\ }} \right)}^{2}}}}</math>


Mit der Feinstrukturkonstanten
Mit der Feinstrukturkonstanten


<math>\gamma \approx \frac{1}{137}</math>
:<math>\gamma \approx \frac{1}{137}</math>


<math>\begin{align}
:<math>\begin{align}


& \lambda =\sqrt{q} \\
& \lambda =\sqrt{q} \\
Zeile 552: Zeile 552:
\end{align}</math>
\end{align}</math>


<math>\begin{align}
:<math>\begin{align}


& \lambda =\sqrt{{{q}^{2}}-{{\gamma }^{2}}}=\sqrt{{{\left( j+\frac{1}{2} \right)}^{2}}-{{\gamma }^{2}}} \\
& \lambda =\sqrt{{{q}^{2}}-{{\gamma }^{2}}}=\sqrt{{{\left( j+\frac{1}{2} \right)}^{2}}-{{\gamma }^{2}}} \\
Zeile 566: Zeile 566:
, so folgt:
, so folgt:


<math>E={{m}_{0}}{{c}^{2}}\left[ 1-\frac{1}{2}{{\left( \frac{\gamma }{\lambda +n\acute{\ }} \right)}^{2}}+\frac{3}{8}{{\left( \frac{\gamma }{\lambda +n\acute{\ }} \right)}^{4}}+O\left( {{\gamma }^{6}} \right) \right]</math>
:<math>E={{m}_{0}}{{c}^{2}}\left[ 1-\frac{1}{2}{{\left( \frac{\gamma }{\lambda +n\acute{\ }} \right)}^{2}}+\frac{3}{8}{{\left( \frac{\gamma }{\lambda +n\acute{\ }} \right)}^{4}}+O\left( {{\gamma }^{6}} \right) \right]</math>


mit
mit


<math>\lambda \left( \gamma  \right)=|q|\sqrt{1-{{\left( \frac{\gamma }{q} \right)}^{2}}}=|q|\left[ 1-\frac{1}{2}{{\left( \frac{\gamma }{q} \right)}^{2}} \right]+O\left( {{\gamma }^{4}} \right)</math>
:<math>\lambda \left( \gamma  \right)=|q|\sqrt{1-{{\left( \frac{\gamma }{q} \right)}^{2}}}=|q|\left[ 1-\frac{1}{2}{{\left( \frac{\gamma }{q} \right)}^{2}} \right]+O\left( {{\gamma }^{4}} \right)</math>


<math>\begin{align}
:<math>\begin{align}


& {{\left( \frac{1}{\lambda +n\acute{\ }} \right)}^{2}}=\frac{1}{{{\left[ n\acute{\ }+|q|-\frac{1}{2}\left( \frac{{{\gamma }^{2}}}{\left| q \right|} \right) \right]}^{2}}}+O\left( {{\gamma }^{4}} \right) \\
& {{\left( \frac{1}{\lambda +n\acute{\ }} \right)}^{2}}=\frac{1}{{{\left[ n\acute{\ }+|q|-\frac{1}{2}\left( \frac{{{\gamma }^{2}}}{\left| q \right|} \right) \right]}^{2}}}+O\left( {{\gamma }^{4}} \right) \\
Zeile 583: Zeile 583:


Setzt man dies in die exakten Energieeigenwerte E ein , so folgt:
Setzt man dies in die exakten Energieeigenwerte E ein , so folgt:
<math>\begin{align}
:<math>\begin{align}
& E={{m}_{0}}{{c}^{2}}\left[ 1-\left( \frac{{{\gamma }^{2}}}{2{{n}^{2}}} \right)-\left( \frac{{{\gamma }^{4}}}{2{{n}^{3}}} \right)\left( \frac{1}{j+\frac{1}{2}}-\frac{3}{4n} \right)+O\left( {{\gamma }^{6}} \right) \right] \\
& E={{m}_{0}}{{c}^{2}}\left[ 1-\left( \frac{{{\gamma }^{2}}}{2{{n}^{2}}} \right)-\left( \frac{{{\gamma }^{4}}}{2{{n}^{3}}} \right)\left( \frac{1}{j+\frac{1}{2}}-\frac{3}{4n} \right)+O\left( {{\gamma }^{6}} \right) \right] \\
& n=1,2,3 \\
& n=1,2,3 \\
Zeile 591: Zeile 591:


<u>'''Diskussion'''</u>
<u>'''Diskussion'''</u>
<math>O\left( {{\gamma }^{0}} \right):E={{m}_{0}}{{c}^{2}}</math>
:<math>O\left( {{\gamma }^{0}} \right):E={{m}_{0}}{{c}^{2}}</math>
Ruheenergie
Ruheenergie
<math>O\left( {{\gamma }^{2}} \right):\Delta {{E}^{(2)}}=-{{m}_{0}}{{c}^{2}}\left( \frac{{{\gamma }^{2}}}{2{{n}^{2}}} \right)=-\frac{{{R}_{H}}}{{{n}^{2}}}</math>
:<math>O\left( {{\gamma }^{2}} \right):\Delta {{E}^{(2)}}=-{{m}_{0}}{{c}^{2}}\left( \frac{{{\gamma }^{2}}}{2{{n}^{2}}} \right)=-\frac{{{R}_{H}}}{{{n}^{2}}}</math>
nicht relativistisches, entartetes Energiespektrum
nicht relativistisches, entartetes Energiespektrum
<math>O\left( {{\gamma }^{4}} \right):\Delta {{E}^{(4)}}=-{{m}_{0}}{{c}^{2}}\left( \frac{{{\gamma }^{4}}}{2{{n}^{3}}} \right)\left( \frac{1}{j+\frac{1}{2}}-\frac{3}{4n} \right)</math>
:<math>O\left( {{\gamma }^{4}} \right):\Delta {{E}^{(4)}}=-{{m}_{0}}{{c}^{2}}\left( \frac{{{\gamma }^{4}}}{2{{n}^{3}}} \right)\left( \frac{1}{j+\frac{1}{2}}-\frac{3}{4n} \right)</math>
Feinstruktur- Aufspaltung. Eine Aufhebung der j-Entartung durch Spin- Bahn- Kopplung.
Feinstruktur- Aufspaltung. Eine Aufhebung der j-Entartung durch Spin- Bahn- Kopplung.
Dabei bleibt die Freiheit der Ausrichtung der Achse des magnetischen Moments, also die <math>2(2j+1)</math>
Dabei bleibt die Freiheit der Ausrichtung der Achse des magnetischen Moments, also die <math>2(2j+1)</math>
Zeile 601: Zeile 601:
- Entartung+ Parität !
- Entartung+ Parität !
====Spektroskopische Beziehung der Feinstrukturterme: <math>n{{l}_{j}}</math>====
====Spektroskopische Beziehung der Feinstrukturterme: <math>n{{l}_{j}}</math>====
<math>n=1:\quad j=\frac{1}{2}:\ 1{{s}_{\frac{1}{2}}}</math>
:<math>n=1:\quad j=\frac{1}{2}:\ 1{{s}_{\frac{1}{2}}}</math>
<math>\begin{array}{*{35}{l}}
:<math>\begin{array}{*{35}{l}}
   {} & n=2:\quad j=\frac{1}{2}:\ 2{{s}_{\frac{1}{2}}}\quad 2{{p}_{\frac{1}{2}}}\quad \quad \quad \quad \quad \quad \quad n\overset{\acute{\ }}{\mathop{\ }}\,=1  \\
   {} & n=2:\quad j=\frac{1}{2}:\ 2{{s}_{\frac{1}{2}}}\quad 2{{p}_{\frac{1}{2}}}\quad \quad \quad \quad \quad \quad \quad n\overset{\acute{\ }}{\mathop{\ }}\,=1  \\
   {} & \quad \quad \quad \,j=\frac{3}{2}:\quad \quad \quad 2{{p}_{\frac{3}{2}}}\quad \quad \quad \quad \quad \quad \quad n\overset{\acute{\ }}{\mathop{\ }}\,=0  \\
   {} & \quad \quad \quad \,j=\frac{3}{2}:\quad \quad \quad 2{{p}_{\frac{3}{2}}}\quad \quad \quad \quad \quad \quad \quad n\overset{\acute{\ }}{\mathop{\ }}\,=0  \\

Version vom 12. September 2010, 16:36 Uhr




In einem rotationssymmetrischen Potenzial haben wir als Dirac- Hamiltonian:

Dabei sind

hermitesche Operatoren

Man kann den Hamilton- Operator schreiben als:

Beweis:

Es gilt weiter:

. Somit existieren gemeinsame Eigenzustände zu

und

Eigenwerte von

Somit:

Schließlich also

Die Eigenwerte von

sind jedoch bekannt, nämlich

mit

Somit:

Es bleibt das radiale Eigenwertproblem für

Geeignete Darstellung für

Für

kann dies durch die Darstellung

mit

erfüllt werden:

Es gilt:

Also

Ansatz für den Radialanteil

Eingesetzt in die Eigenwertgleichung für H:

folgt:

Also:

Skalentransformation:

Führt man des weiteren ein:

Also einen skalierten Radius und die Feinstrukturkonstante,

wodurch sich auch das Potenzial vereinfacht zu:

Randbedingung:

regulär bei

für

Betrachte

also gebundene Zustände

Asymptotisches Verhalten:

Weil

divergiert !

Ansatz:

Es existieren nichttriviale Lösungen

, falls

Also

und regulär bei

Ansatz:

Die Lösung erfolgt über einen Potenzreihenansatz:

usw... wird dies ebenfalls für

aufgestellt

Koeffizientenvergleich liefert:

bis auf Normfaktor

k=0,1,2,.... Rekursionsformel !!

Verhalten für große k:

Dies kann man einsetzen in

und es folgt:

Falls die Potenzreihen

nicht abbrechen, so divergiert

exponentiell für

Dies ist jedoch ein Widerspruch zu den gesetzten Randbedingungen !

Also muss es einen Abbruch bei

geben:

Setzt man dies in die Rekursionsformel ein, so folgt:

Diese beiden Gleichungen stimmen jedoch für alle f,g überein, da

Setzt man

in

ein, so folgt mit

Weiter gilt:

Löst man dies nach den exakten Energieeigenwerten, die sich damit ergeben, also nach E auf, so erhält man die Feinstrukturformel:

Mit der Feinstrukturkonstanten

entwickelt man die Energieeigenwerte nach der Feinstrukturkonstanten bis

, so folgt:

mit

Setzt man dies in die exakten Energieeigenwerte E ein , so folgt:

Diskussion

Ruheenergie

nicht relativistisches, entartetes Energiespektrum

Feinstruktur- Aufspaltung. Eine Aufhebung der j-Entartung durch Spin- Bahn- Kopplung. Dabei bleibt die Freiheit der Ausrichtung der Achse des magnetischen Moments, also die - fache - Entartung+ Parität !

Spektroskopische Beziehung der Feinstrukturterme:

n´=0 ..